Sorry for the delay guys, but I will be up with a SC project soon
Here are the OEs for the weaken set
"WEAKEN" SET OFFICIAL EXPLANATIONS
WEAKEN
1.
On average, the team hit more home runs playing in front of larger crowds than
in front of smaller crowds. The argument attributes this statistic to the motivation
that comes from playing in front of larger crowds. In order to undermine this
conclusion, look for another reason to explain why more home runs were hit in
front of larger crowds.
(A) The argument makes a claim about the collective behavior of the team. This
collective claim does not preclude certain individuals from hitting fewer home
runs in larger stadiums.
(B) The claim made in the argument is based on the size of the crowd in each
stadium. For whom the fans cheered is irrelevant to the argument.
(C) Similar to answer choice A, this choice cites one specific example of
contradictory information, while the argument is based on the average behavior
of the team throughout the entire season. The does not strongly undermine that,
on average, the team was motivated by larger crowds.
(D) CORRECT. This choice explains that the larger stadiums actually have
different dimensions from the smaller stadiums. In order to accommodate a
larger number of fans, the outfield walls are closer to the batters. Thus, it is very
possible that the greater number of home runs is due to the fact that the ball
does not have to travel as far in larger stadiums.
(E) The announcer’s opinion is not relevant to the argument, and, even if it were,
this choice would strengthen the argument.
2.
The conclusion is that "if major industries increase their capital reserves, the
employment rate will not decline in the future." Why? Because major industry did
not have capital reserves. The author assumes that having capital reserves is
sufficient to prevent a decline in the employment rate. We are asked to cast
doubt (i.e., weaken) the author's claim.
(A) Whether the drop in employment was foreseen does not relate to the core of
the argument, which is that capital reserves will prevent another decline in the
employment rate.
(B) The fact that some major industries had appreciable capital reserves does
not contradict the claim that an increase in these reserves would prevent a future
drop in employment rates.
(C) CORRECT. The author neglects to take into account the fact that other
factors, such as an increase in labor costs, could adversely affect the
employment rate. For example, if the cost of labor becomes prohibitively
expensive, even with increased reserves, the employment rate could decline.
(D) Legislation mandating a certain level of reserves does not contradict the
claim that increased reserves would prevent a drop in employment rates.
(E) The fact that the employment rate was more severe this year than last does
not contradict the claim that an increase in reserves would prevent a drop in the
employment rate.
3.
Farmers in developing countries claim that the global price of wheat is low
because American farmers produce too much of the grain. They also claim that
American farmers produce too much wheat because they have no incentive to
manage their crops, since the U.S. government will buy whatever wheat
American farmers cannot sell on the open market. We are asked to find a choice that
weakens the claims of the farmers in developing countries that removing the
American subsidy would cause the price of wheat to rise.
(A) That there are uses for wheat that is not eaten is irrelevant here. This does
not address any aspect of the farmers' claims.
(B) The fact that buyers of wheat can predict their needs in advance is irrelevant
here, because the text indicates that American farmers do not pay attention to
actual demand for wheat.
(C) In this argument, the global market for soybeans is irrelevant to the global
market for wheat, which is a different commodity with different demand, supply,
and pricing structures.
(D) CORRECT. The farmers assume that the sole cause of the wheat surplus is
the United States. This answer choice suggests that other countries would
modify their output to counterbalance any reduction on the part of the United
States, keeping prices constant instead of allowing them to rise.
(E) The price of another crop is largely irrelevant. Moreover, the fact that the
price of sorghum, a non-subsidized crop, is lower tends to support, rather than
weaken, the claims of the farmers.
4.
This argument concludes that “Hollywood studios have little chance of making
money this year by exporting their films worldwide,” based on the premise that
profits from piracy overseas rose sharply last year. The argument assumes that
no other relevant conditions have changed for the better since then. The question
asks for a choice that does not weaken the argument, which means that the
correct answer will either strengthen it or is irrelevant.
(A) This choice weakens the argument. It attacks the necessary assumption that
$500 million is a large enough amount to destroy Hollywood profits.
(B) This choice weakens the argument. It attacks the necessary assumption that
there would not be a heightened and effective law enforcement campaign against
piracy.
(C) This choice weakens the argument. It attacks the assumption that there are
no other ways for Hollywood studios to profit from export of its films than DVDs.
(D) CORRECT. This choice is irrelevant. The conclusion (and premise) concern
money. That all of the Academy Award-nominated films were pirated last year
does not increase or diminish the probability of Hollywood studios making money
this year. Thus, this does not weaken the argument.
(E) This choice makes the conclusion less likely by providing a premise that
suggests that this year’s profits internationally will rise.
5.
The analyst argues that the mayoral candidate who opposes the deportation plan
will win the governor’s race because 60% of city residents also oppose the plan.
The analyst assumes that a majority of residents will vote for this candidate
based on his position on illegal immigration. Any statement that calls this
assumption into question will weaken the argument. We are looking for the one
statement that does NOT call this assumption into question.
(A) This statement calls into question the assumption that voters will cast their
ballots based on the illegal immigration issue. Therefore, this statement
weakens the analyst's argument.
(B) CORRECT. This does not weaken the argument. In fact, if some of those
who support the plan are willing to reconsider, they may ultimately oppose the
original plan and decide to vote for the candidate who is also in opposition. If
anything, this would help justify the analyst's claim that the candidate who
opposes the plan will win the election.
(C) This statement calls into question the assumption that a majority of residents
will vote for the candidate who opposes the plan. If many of these residents are
not registered voters, they will not be able to vote, regardless of their position on
the immigration issue. This weakens the argument.
(D) This calls into question the assumption that the residents will vote based on
the illegal immigration issue. This statement shows that voters have a history of
voting for the incumbent despite his controversial position on important issues. It
is possible that the voters will again vote for the incumbent, even if he has taken
an unpopular position on the illegal immigration issue. This weakens the
argument.
(E) If just under 30% of the residents are illegal immigrants, it is likely that many
of the 60% in opposition to the plan are actually illegal immigrants themselves. If
these people can’t vote, it is less likely that the candidate who opposes the plan
will win.
6.
Adam concludes that to cover the $4,000 cost of a booth at the fair, he will need
to sell 400 sandwiches at $10 each. He relies on information about the number of
customers served by the average booth in previous years. The question asks
what most weakens Adam’s conclusion. The correct answer will either show that
Adam has overlooked other important costs, or that his logic is fundamentally flawed.
There could be more than one choice that seems to suggest that Adam
might not break even; the correct answer will be the one that eliminates any
possibility that he will break even.
(A) Though this is certainly true in a practical sense, it does not show that Adam's
plan will not work and that he will not stay within his budget. In fact, if he could
sell more than 400 sandwiches, it is likely that this would help him break even.
(B) CORRECT. Each sandwich costs some amount of money to make and sell,
so even if Adam sells the 400 sandwiches as planned, his net income will
certainly be less than $4,000. Based on this statement, Adam definitely will not
break even.
(C) Though it is true that an average number of customers means some booths
served more and some served fewer, this statement does not tell us that Adam
will have fewer than 400 or that his plan cannot work. Consider also that each
customer might buy more than one sandwich, so Adam could have fewer
customers but still sell enough sandwiches to cover his costs.
(D) Although some people will buy other types of food at the fair, this statement
fails to conclusively weaken Adam’s logic. Adam relied on information about the
average number of customers for food booths in previous years, but those
booths may have sold sandwiches or other foods – we simply don’t have enough
information to judge whether this statement strengthens or weakens Adam’s
conclusion.
(E) Sharing the booth would actually make it more likely that Adam would break
even. If he shares the cost of the booth rental, then he is more likely to sell
enough sandwiches to cover his costs.
7.
The argument concerns the economic impact on restaurants in Prohibitionland if
the service of alcoholic beverages is banned. It presents evidence that, despite
restrictions on the service of alcohol in certain areas of Prohibitionland, sales
taxes in restaurants in those areas rose at a higher rate than for those in other
parts of Prohibitionland, suggesting that the ban would not have any adverse
economic impact. We are asked to support the restaurant proprietors' claim , so
the correct answer choice will call the relevance of the seemingly contradictory
evidence into question.
(A). This answer choice may seem to strengthen the argument that banning the
service of alcoholic beverages would have an adverse impact on restaurants.
However, as the evidence involves data for the entire year, citing a short-term
negative impact on restaurant visitation at the beginning of the year does not
measurably strengthen the argument.
(B) The relative tax rate on food and beverages as compared to other consumer
good is irrelevant here.
(C) A gradual decline in alcohol consumption over the past 20 years would
suggest that over time, any ban on alcohol would have an increasingly small
impact on restaurant visitation, weakening the proprietors’ argument.
(D) CORRECT. This statement calls the evidence into question by indicating that
any measured increase in sales taxes and, presumably, revenues for restaurants
that have been operating under the restrictions last year enacted is irrelevant, as
the restrictions could be argued to be completely different than the total ban that
is being proposed. This answer choice substantially strengthens the proprietors’
argument by threatening to make the cited evidence irrelevant.
(E) The fact that overall sales tax revenue did not increase at a higher rate in the
provinces that enacted the restrictions on alcoholic beverages weakens the
proprietors’ argument, as it makes the cited evidence more compelling by ruling
out the possibility of different growth rates in the different areas.
8.
The official's conclusion is that people who claim that the U.S. is more vulnerable
than other nations because of the country's lack of a national vaccine laboratory
are disloyal and incorrect. His basis for that conclusion is that the U.S. has
generally long life span and low infant mortality relative to all UN countries.
Moreover, he cites the high quality of American hospitals, and he adds that many
people he knows from around the world come to the U.S. for medical care. We
are asked to find the choice that does NOT point out a weakness or potential
weakness in the official's argument.
(A) CORRECT. This choice states that the high quality of hospitals is not a factor
affecting the public's vulnerability to infectious disease. However, the quality of
hospitals very arguably does affect how vulnerable the public is to such disease.
(B) This choice highlights the official's logical jump from "disloyal" to "wrong" in
the phrase "these critics are disloyal and thus wrong about the public's
vulnerability." There is no necessary connection between disloyalty and
wrongness.
(C) If the Europeans that the official cited overwhelmingly consist of wealthy men
over the age of fifty, then the official relied on an unrepresentative sample to
justify his claim. What is true of wealthy older European men is not necessarily
true of Europeans or non-Americans generally.
(D) If the average life span of Americans is determined by causes other than
infectious disease, then the official is not limiting his evidence to cases relating to
vaccines. Other causes of death are not relevant to the critics' argument.
(E) The ranking of the United States relative to all UN countries is misleading,
since it does not compare the U.S. to other "advanced industrialized" or
"developed" nations, as the critics' claim does.
9.
The conclusion of the argument is that insurance companies do not have a
significant economic incentive to delay claim payments to doctors. To weaken
this conclusion, an answer choice must provide some significant economic
incentive for insurance companies to be tardy in paying doctors for legitimate
medical claims.
(A) While the fact that some doctors who submit accurate bills to insurance
companies still receive tardy payments seems to indicate that there must be
something other than errors causing delayed payments, it fails to prove that the
insurance company has an economic incentive to deliberately delay claim
payments to doctors. For example, this fact could simply indicate that the
insurance companies are inefficient at handling all of their paperwork.
(B) This choice compares costs insurance companies must absorb due to
incorrect bills to costs physicians must absorb due to tardy payments. However,
this information is irrelevant to establishing an economic incentive for insurance
companies to delay claim payments to doctors.
(C) The argument is focused on the payment of legitimate claims; the rising
proportion of illegitimate claims does not establish a clear economic incentive for
insurance companies to delay payments of legitimate claims.
(D) The types of billing errors made by doctors' offices does not establish any
economic motive for insurance companies to make a practice of delaying
payments to doctors.
(E) CORRECT. This choice articulates a logical chain that establishes a clear
economic motive for insurance companies to be tardy in paying doctors for
legitimate medical claims. If insurance companies delay payments to doctors,
this results in a 10 percent increase in overhead costs for physicians. These
costs ultimately result in higher fees that doctors charge to insurance companies.
Insurance companies, in turn, raise the premiums they charge consumers for
health coverage. This choice states that the insurance companies increase their
fees to consumers far more than the doctors increase their fees to insurance
companies, enabling the insurance companies to pocket the difference; therein
lies the economic motive for insurance companies to be tardy in paying doctors
for legitimate medical claims. 10.
This argument concludes that a decline in the percentage of retirees who
relocate to SunState will have a negative impact on businesses there that cater
to retirees. However, a decline in this percentage would only have a negative
impact on businesses if it indicated a decrease in the actual number of retirees. If
the actual number of retirees is steady or increasing, then a decrease in the
percentage wouldn't matter. As we are looking for a statement that weakens the
argument, we should look for an answer choice that somehow mitigates the
effect of this percentage decrease.
(A) The fact that SunState attracts more retirees than any other state does not
address the impact of the declining proportion of retirees moving to SunState.
(B) The existence of other businesses in SunState that do not cater to retirees is
not relevant.
(C) Any increase in departure of retirees from SunState to accept re-employment
would further damage businesses that serve retirees. However, the argument
explicitly discusses the impact of the declining percentage of retirees relocating
to SunState, and no other factors, making this answer choice irrelevant. In any
case, this answer choice suggests that such businesses will indeed lose
business, which would strengthen the conclusion, not weaken it.
(D) Low property taxes provide one reason why SunState is an appealing
destination for retirees, but this is not relevant in determining the economic
impact of the smaller proportion of retirees moving to SunState overall.
(E) CORRECT. If the total number of retirees that relocated to other states
increased significantly, a 10 percent reduction in the proportion of retirees that
moved to SunState may not result in a reduction in the actual number of people
who moved to SunState. This choice weakens the contention that businesses
that cater to retirees in SunState will suffer from a drop-off resulting from the
percentage decrease.